Cartridge Loading for a phono pre amp


Hello,

I have recently acquired a phono pre amp recommended by Michael Fremer.  It is “THE VINYL”, from QHW audio, Spain.  It got a great review.  I have a Benz Micro Glider rated at 1.1MV.  I have no idea how to set the dip switches for MC Load impedance for this cartridge. The options I have are as follows: 47K, 1K, 560R, 470R, 100R, and 47R.  I have a solid state amp and pre-amp, and also have a sub that I use, rarely.

Any advice would be most appreciated!!

judsauce

@rauliruegas I see you are refusing to answer my prior question to you. I've handed all the information you need to you on a silver platter. But the way you are reacting suggests you are not here for any other reason than to troll. If that is not the case, go ahead and answer my prior question. Here again for your convenience, are both of my prior questions.   If you are disinclined, I can only assume its because trolling is your goal rather than anything else:

Anyone with an elementary school education can work out the math here. Perhaps, knowing that, you can tell me where that current is coming from??  If you can answer that, a bonus question: what is the consequence of that current flow?

 

Do it with this example:

tonearms SME 5   Cartridge: Lyra Kleos    resonance frequency: 9hz.

 

So you can go from 47k to 100 oms  and 70 pf of capacitance

 

Now, show that that load change puts  out of the ideal range. Easy for you, just show it.

So you're not going to answer my question? That leads me to think you are simply trolling. Because there are internal losses, the cantilever will not be quite 470 times harder to move; we'll call it 400, assuming that its also designed to operate at 47K. 

This is based simply on this fact:

https://physics.stackexchange.com/questions/120005/why-do-electricity-generators-have-to-work-harder-for-higher-loads

Since the generator only converts mechanical power to electric power, the motor driving the generator shaft must supply at least 10 times more power in order for the generator to supply 10 times more power.

This is fundamental and inescapable.

You are correct on one thing, it was pretty easy.

The arm has an 11 gr effective mass, the cartridge is 9 grams; the compliance is rated 'Approx. 12x10-6cm/dyne at 100Hz' note the word 'Approx' (that is variable based on load). Now plug those values into the calculator:

http://korfaudio.com/calculator

You'll see for starters this cartridge is not a good choice for this arm. Now let's assume that the cartridge is loaded at 800 Ohms. By decreasing the load to 100 Ohms we ask slightly less than 8x the current out of the cartridge. This will increase the stiffness a bit, how much is difficult to know without more specific information about the cartridge, but as you see from the first link of this post it has to be significant. Let's assume that the load is a fractional bit of the compliance instead of dominating it. You can see what happens if the compliance is decreased by decrementing from 12 to 11, which is conservative; this puts the results outside of the green zone on both charts.

 

@holmz Oh man, you're doing Raul's homework for him😄

If I cannot get fed, then at least I can do the cooking or the dishes ;)

This has been a fabulous technical lesson provided by one of the industries best. This has also been an exercise in personal credibility. Desperation vs confidence. Compliance vs torque. What a discussion; one for the ages.